Extrinsic curvature of Kerr-Schild using ADM equations

  • Thread starter ergospherical
  • Start date
  • #1
ergospherical
1,019
1,299
Homework Statement
Exercise 2.33 of Shapiro & Baumgarte. In Kerr-Schild coords, ##ds^2 = (\eta_{ab} + 2H l_a l_b) dx^a dx^b## where the null vector ##l_a## has Cartesian components ##l_t = 1## and ##l_i = x^i/r## and ##H := M/r##. Identify the lapse, shift, spatial metric and show that the extrinsic curvature is$$K_{ij} = \frac{2H\alpha}{r}(\delta_{ij} - (2+H)l_i l_j)$$
Relevant Equations
ADM equations
I can think of a couple of ways to go about determining the extrinsic curvature, but the most direct seems to be straight from the ADM equation for the evolution of the spatial metric,$$\partial_t \gamma_{ij} = \beta^m \partial_m \gamma_{ij} + \gamma_{m(i} \partial_{j)} \beta^m - 2\alpha K_{ij}$$From the form of the metric in coordinates adapted to the 3+1 split, it's easy to write down the lapse ##\alpha = 1##, shift ##\beta^i = 2Hx^i/r## and the spatial metric ##\gamma_{ij} = \delta_{ij} + 2H x^i x^j/r^2##. I find the following terms:\begin{align*}
\beta^m \partial_m \gamma_{ij} &= -4H^2 l_i l_j \\
\gamma_{mi} \partial_j \beta^m &= \frac{2H}{r}(\delta_{ij} - 2(2+H)l_i l_j)
\end{align*}The spatial metric being time independent means that$$2\alpha K_{ij} = \beta^m \partial_m \gamma_{ij} + \gamma_{m(i} \partial_{j)} \beta^m$$The expressions computed above don't give the quoted result for ##K_{ij}## (as far as I can tell). Can anyone spot my error?
 
Physics news on Phys.org
  • #2
I can't really address your question without refreshing my memory on ADM, which currently I don't have the time for. But reading it, I (think I) can see some things:

ergospherical said:
##\gamma_{ij} = \delta_{ij} + 2H x^i x^j/r^2##
Seriously? :smile:

##x_i=g_{i \nu}x^\nu## . Did you remember to account for that in the differentiation ##\frac{\partial x_i}{\partial x^j}## ?

ergospherical said:
##\beta^m \partial_m \gamma_{ij} = -4H^2 l_i l_j ##
It appears as not having the same units as the line below and the textbook's solution.

One more: even if ##\alpha = 1##, its appearance once as proportional to ##K_{i j} ## and once as inverse, seems odd (superficially).
 
  • #3
Because ##l^a## is a null vector (with respect to both ##\eta## and ##g##), then you can check that ##l_i = l^i = x^i/r##. So ##g_{ij} = \delta_{ij} + 2H l_i l_j = \delta_{ij} + 2H x^i x^j/r^2##, with indices up in the last term...
 
  • #4
I looked it in the books. You are right.

If ##\beta##, ##H## and ##\gamma_{ij}## are unitless (as you presented them), then ##\beta^m \partial_m \gamma_{ij}## should be of units 1/length
 
  • Like
Likes ergospherical
  • #5
Yeah, the dimensions aren't right. There must be a sloppy mistake somewhere in my work -- including possibly some missed raised/lowered indices. I'll look tomorrow!
 
  • #6
You might find table 2.1 on page 50 interesting
 
  • Like
Likes ergospherical
  • #7
JimWhoKnew said:
You might find table 2.1 on page 50 interesting
For sure! So I have the completely wrong lapse (and contra-variant shift). They get, ##\alpha^2 = (1 + \tfrac{2M}{r})^{-1}## and ##\beta^i = \tfrac{2M}{r} \alpha^2 l^i##. I've checked it's indeed consistent with the general form of the metric in ADM-adapted coordinates, e.g. \begin{align*}
\beta_i = \gamma_{ij} \beta^j &= [\delta_{ij} + \tfrac{2M}{r}l_i l_j] \tfrac{2M}{r} \alpha^2 l^j \\
&= \tfrac{2M}{r} \alpha^2 l_i \left( 1 + \tfrac{2M}{r} l^j l_j \right) \\
&= \tfrac{2M}{r} l_i \\
&= g_{0i}
\end{align*}I guess it's not supposed to be obvious that this is the correct decomposition, which is why they've given it in the table.
 

Similar threads

  • Advanced Physics Homework Help
Replies
0
Views
1K
  • Differential Geometry
Replies
0
Views
298
  • Advanced Physics Homework Help
Replies
2
Views
1K
  • Advanced Physics Homework Help
Replies
3
Views
1K
  • Advanced Physics Homework Help
Replies
1
Views
1K
  • Advanced Physics Homework Help
Replies
1
Views
1K
  • Advanced Physics Homework Help
Replies
3
Views
969
  • Advanced Physics Homework Help
Replies
1
Views
1K
  • Advanced Physics Homework Help
Replies
29
Views
3K
  • Cosmology
Replies
9
Views
1K
Back
Top